www.vorhilfe.de
- Förderverein -
Der Förderverein.

Gemeinnütziger Verein zur Finanzierung des Projekts Vorhilfe.de.
Hallo Gast!einloggen | registrieren ]
Startseite · Mitglieder · Impressum
Forenbaum
^ Forenbaum
Status VH e.V.
  Status Vereinsforum

Gezeigt werden alle Foren bis zur Tiefe 2

Navigation
 Startseite...
 Suchen
 Impressum
Das Projekt
Server und Internetanbindung werden durch Spenden finanziert.
Organisiert wird das Projekt von unserem Koordinatorenteam.
Hunderte Mitglieder helfen ehrenamtlich in unseren moderierten Foren.
Anbieter der Seite ist der gemeinnützige Verein "Vorhilfe.de e.V.".
Partnerseiten
Weitere Fächer:

Open Source FunktionenplotterFunkyPlot: Kostenloser und quelloffener Funktionenplotter für Linux und andere Betriebssysteme
Forum "Lineare Algebra - Moduln und Vektorräume" - Polynom- und Hauptidealring
Polynom- und Hauptidealring < Moduln/Vektorraum < Lineare Algebra < Hochschule < Mathe < Vorhilfe
Ansicht: [ geschachtelt ] | ^ Forum "Lineare Algebra - Moduln und Vektorräume"  | ^^ Alle Foren  | ^ Forenbaum  | Materialien

Polynom- und Hauptidealring: Beweis richtig?
Status: (Frage) beantwortet Status 
Datum: 20:14 Do 10.05.2012
Autor: Blackburn4717537

Aufgabe
Sei K ein Körper.

Zeigen Sie: K[t] ist ein Hauptidealring.

Hallo,

könnte bitte jemand drüber gucken, ob mein Beweis richtig ist?

Beweis:

Sei K Körper.

Sei I [mm] \subseteq [/mm] K[t] ein Ideal.

Zeige: [mm] \exists [/mm] g [mm] \in [/mm] K[t] : <g> = I

1. Fall: I ist das Nullideal [mm] \Rightarrow [/mm] I ist Hauptideal.
2. Fall: I ist 1-Ideal [mm] \Rightarrow [/mm] I ist Hauptideal.
3. Fall: Sei I nun weder Null- noch 1-Ideal.

Wähle g [mm] \in [/mm] I [mm] \setminus [/mm] {0} mit deg(g) = min{deg(i) | i € I \ {0} }.
[mm] \Rightarrow [/mm] deg(g) [mm] \le [/mm] deg(i) [mm] \forall [/mm] i € I \ {0}

Sei i € I \ {0} beliebig.
Wende Division mit Rest an:

[mm] \exists [/mm] q, r € K[t]: i = q * g + r mit deg(r) < deg(g)

Zeige r = 0 [mm] \gdw [/mm] deg(r) = - [mm] \infty [/mm]

[mm] \Rightarrow [/mm] r = i - q * g € I

Da r € I und g € I \ {0} (mithin also deg(g) [mm] \not= [/mm] - [mm] \infty [/mm] ) und deg(r) < deg(g) und deg(g) minimal:

[mm] \Rightarrow [/mm] deg(r) = - [mm] \infty [/mm]
[mm] \Rightarrow [/mm] r = 0
[mm] \Rightarrow [/mm] i = q * g

Da nun i € I beliebig, und g € I \ {0} fest, und nach der Division mit Rest immer ein solches q existiert, sodass die obige Gleichung erfüllt ist, wird I von g erzeugt, also <g> = I.

[mm] \Rightarrow [/mm] K[t] ist Hauptidealring.

        
Bezug
Polynom- und Hauptidealring: Antwort
Status: (Antwort) fertig Status 
Datum: 20:44 Do 10.05.2012
Autor: Schadowmaster

moin,

Bevor wir zu deinem Beweis kommen erstmal eine Sache:
Du wendest hier den euklidischen Algorithmus auf $K[x]$ an.
Ringe, in denen ein solcher existiert bzw. möglich ist, werden Euklidische Bereiche genannt. Man kann zeigen, dass jeder Euklidische Bereich ein Hauptidealbereich ist - habt ihr das vielleicht schon in der Vorlesung gemacht?

Dann mal zu deinem Beweis:

> Sei K ein Körper.
>
> Zeigen Sie: K[t] ist ein Hauptidealring.
>  Hallo,
>  
> könnte bitte jemand drüber gucken, ob mein Beweis richtig ist?
>  
> Beweis:
>  
> Sei K Körper.
>  
> Sei I [mm]\subseteq[/mm] K[t] ein Ideal.
>  
> Zeige: [mm]\exists[/mm] g [mm]\in[/mm] K[t] : <g> = I
>  
> 1. Fall: I ist das Nullideal [mm]\Rightarrow[/mm] I ist Hauptideal.
>  2. Fall: I ist 1-Ideal [mm]\Rightarrow[/mm] I ist Hauptideal.
>  3. Fall: Sei I nun weder Null- noch 1-Ideal.
>  
> Wähle g [mm]\in[/mm] I [mm]\setminus[/mm] {0} mit deg(g) = min{deg(i) | i € I \ {0} }.

Begründe vielleicht kurz, wieso ein solches $g$ existiert.

>  [mm]\Rightarrow[/mm] deg(g) [mm]\le[/mm] deg(i) [mm]\forall[/mm] i € I \ {0}

allgemein einmal an dieser Stelle: $i$ ist meist eine Laufvariable. Polynome werden allgemein meist mit $f,g$ oder $p,q$ bezeichnet.
Es ist nicht falsch $i$ zu wählen, aber es ist ungewöhnlich.

> Sei i € I \ {0} beliebig.
>  Wende Division mit Rest an:
>  
> [mm]\exists[/mm] q, r € K[t]: i = q * g + r mit deg(r) < deg(g)

Das ist keine "Anwendung" der Div. mit Rest, die Existenz folgt aus der Tatsache, dass eine solche möglich ist (eben da $K[x]$ ein Euklidischer Bereich ist).
Wenn du von "anwenden" sprichst dann meint das meist eine konstruktive Anwendung, hier gibst du $q,r$ ja nicht an sondern begründest nur, warum es solche geben muss.


> Zeige r = 0 [mm]\gdw[/mm] deg(r) = - [mm]\infty[/mm]

Hmm, willst du das wirklich zeigen?
Das ist nämlich eine Definition, die könnte schwer zu zeigen sein.^^

> [mm]\Rightarrow[/mm] r = i - q * g € I

> Da r € I und g € I \ {0} (mithin also deg(g) [mm]\not=[/mm] - [mm]\infty[/mm] ) und deg(r) < deg(g) und deg(g) minimal:
>  
> [mm]\Rightarrow[/mm] deg(r) = - [mm]\infty[/mm]
>  [mm]\Rightarrow[/mm] r = 0
>  [mm]\Rightarrow[/mm] i = q * g
>  
> Da nun i € I beliebig, und g € I \ {0} fest

"nun $i [mm] \in [/mm] I$ beliebig" klingt so, als wäre $i$ davor fest gewesen.
Also lass das "nun" besser weg...

> , und nach der Division mit Rest immer ein solches q existiert, sodass die obige Gleichung erfüllt ist, wird I von g erzeugt, also <g> = I.
>  
> [mm]\Rightarrow[/mm] K[t] ist Hauptidealring.

Ah, ok, du willst sagen, dass aus Gradgründen $r=0$ gelten muss.
Der Beweis sieht ganz gut aus, am Schluss wo du mit "$r = 0 [mm] \gdw$" [/mm] angesetzt hast wären vielleicht 1-2 kleine erläuternde Worte ganz nett.
Und wie gesagt würde ich mal nachforschen, ob ihr "Euklidischer Bereich [mm] $\Rightarrow$ [/mm] Hauptidealbereich" schon irgendwo hattet.

lg

Schadow

Bezug
                
Bezug
Polynom- und Hauptidealring: Frage (beantwortet)
Status: (Frage) beantwortet Status 
Datum: 21:11 Do 10.05.2012
Autor: Blackburn4717537


> moin,
>  
> Bevor wir zu deinem Beweis kommen erstmal eine Sache:
>  Du wendest hier den euklidischen Algorithmus auf [mm]K[x][/mm] an.
>  Ringe, in denen ein solcher existiert bzw. möglich ist,
> werden Euklidische Bereiche genannt. Man kann zeigen, dass
> jeder Euklidische Bereich ein Hauptidealbereich ist - habt
> ihr das vielleicht schon in der Vorlesung gemacht?

Hi,

euklidische Bereiche hatten wir noch nicht gehabt.

>  
> Dann mal zu deinem Beweis:
>  > Sei K ein Körper.

> >
> > Zeigen Sie: K[t] ist ein Hauptidealring.
>  >  Hallo,
>  >  
> > könnte bitte jemand drüber gucken, ob mein Beweis richtig ist?
>  >  
> > Beweis:
>  >  
> > Sei K Körper.
>  >  
> > Sei I [mm]\subseteq[/mm] K[t] ein Ideal.
>  >  
> > Zeige: [mm]\exists[/mm] g [mm]\in[/mm] K[t] : <g> = I
>  >  
> > 1. Fall: I ist das Nullideal [mm]\Rightarrow[/mm] I ist Hauptideal.
>  >  2. Fall: I ist 1-Ideal [mm]\Rightarrow[/mm] I ist Hauptideal.
>  >  3. Fall: Sei I nun weder Null- noch 1-Ideal.
>  >  
> > Wähle g [mm]\in[/mm] I [mm]\setminus[/mm] {0} mit deg(g) = min{deg(i) | i € I \ {0} }.
>  
> Begründe vielleicht kurz, wieso ein solches [mm]g[/mm] existiert.

Wenn I ein Ideal ist, aber ungleich dem Null- und 1-Ideal, muss es ein solches g geben, denn Ideale sind ja auch ungleich der leeren Menge, oder was meinst du?

>  
> >  [mm]\Rightarrow[/mm] deg(g) [mm]\le[/mm] deg(i) [mm]\forall[/mm] i € I \ {0}

>  
> allgemein einmal an dieser Stelle: [mm]i[/mm] ist meist eine Laufvariable. Polynome werden allgemein meist mit [mm]f,g[/mm] oder [mm]p,q[/mm] bezeichnet.
>  Es ist nicht falsch [mm]i[/mm] zu wählen, aber es ist ungewöhnlich.
>  

Ja stimmt, keine Ahnung, warum ich i gewählt habe.^^

> > Sei i € I \ {0} beliebig.
>  >  Wende Division mit Rest an:
>  >  
> > [mm]\exists[/mm] q, r € K[t]: i = q * g + r mit deg(r) < deg(g)
>  
> Das ist keine "Anwendung" der Div. mit Rest, die Existenz folgt aus der Tatsache, dass eine solche möglich ist (eben da [mm]K[x][/mm] ein Euklidischer Bereich ist).
>  Wenn du von "anwenden" sprichst dann meint das meist eine konstruktive Anwendung, hier gibst du [mm]q,r[/mm] ja nicht an sondern begründest nur, warum es solche geben muss.
>  

Also schreibe ich: Wegen der Division mit Rest ex. solche q,r... ?

>
> > Zeige r = 0 [mm]\gdw[/mm] deg(r) = - [mm]\infty[/mm]
>  
> Hmm, willst du das wirklich zeigen?
>  Das ist nämlich eine Definition, die könnte schwer zu zeigen sein.^^
>  

Ich will zeigen, dass r das Nullpolynom sein muss, bzw. äquivalent dazu, dass deg(r) = - [mm] \infty [/mm] ist.

> > [mm]\Rightarrow[/mm] r = i - q * g € I
>  
> > Da r € I und g € I \ {0} (mithin also deg(g) [mm]\not=[/mm] - [mm]\infty[/mm] ) und deg(r) < deg(g) und deg(g) minimal:
>  >  
> > [mm]\Rightarrow[/mm] deg(r) = - [mm]\infty[/mm]
>  >  [mm]\Rightarrow[/mm] r = 0
>  >  [mm]\Rightarrow[/mm] i = q * g
>  >  
> > Da nun i € I beliebig, und g € I \ {0} fest
>  
> "nun [mm]i \in I[/mm] beliebig" klingt so, als wäre [mm]i[/mm] davor fest gewesen.
>  Also lass das "nun" besser weg...

Ok.

>  
> > , und nach der Division mit Rest immer ein solches q existiert, sodass die obige Gleichung erfüllt ist, wird I von g erzeugt, also <g> = I.
>  >  
> > [mm]\Rightarrow[/mm] K[t] ist Hauptidealring.
>
> Ah, ok, du willst sagen, dass aus Gradgründen [mm]r=0[/mm] gelten muss.
>  Der Beweis sieht ganz gut aus, am Schluss wo du mit "[mm]r = 0 \gdw[/mm]" angesetzt hast wären vielleicht 1-2 kleine erläuternde Worte ganz nett.

Ok.

>  Und wie gesagt würde ich mal nachforschen, ob ihr "Euklidischer Bereich [mm]\Rightarrow[/mm] Hauptidealbereich" schon irgendwo hattet.
>  

Danke schonmal, dass du drüber geguckt hast! :)

> lg
>  
> Schadow


Bezug
                        
Bezug
Polynom- und Hauptidealring: Antwort
Status: (Antwort) fertig Status 
Datum: 18:01 Fr 11.05.2012
Autor: Schadowmaster


> Wenn I ein Ideal ist, aber ungleich dem Null- und 1-Ideal, muss es ein solches g geben, denn Ideale sind ja auch ungleich der leeren Menge, oder was meinst du?

Was ich eigentlich meinte ist die Frage, wieso deine betrachtete Menge so ein Minimum hat.
Nimmst du zB das Intervall $(0,1)$ als Teilmenge der reellen Zahlen, so hat dieses weder Maximum noch Minimum.
Und auch die Gradmenge eines Ideals ist (meist^^) unendlich groß, also könntest du hier ggf. noch kurz erwähnen, wieso es ein solches Minimum dennoch gibt.
Vor allem wenn ihr passende Sätze zu dem Thema schon hattet; falls nicht sollte es aber auch so gehen.

Bezug
                                
Bezug
Polynom- und Hauptidealring: Frage (beantwortet)
Status: (Frage) beantwortet Status 
Datum: 22:55 So 13.05.2012
Autor: Blackburn4717537

Hm, stimmt. Dein Beispiel ist ganz gut, weil ich jetzt gerade nicht mehr so sicher bin, warum man es überhaupt so ein Minimum gibt. Sätze dazu hatten wir noch nicht gehabt. Ich hatte, das jetzt auch nur so gemacht, weil ein anderer Beweis, den wir gemacht haben, ähnlich war.

Bezug
                                        
Bezug
Polynom- und Hauptidealring: Antwort
Status: (Antwort) fertig Status 
Datum: 10:02 Mo 14.05.2012
Autor: Schadowmaster

Das Minimum existiert, weil die natürlichen Zahlen wohlgeordnet sind.
Das heißt (passenderweise^^), dass jede nichtleere Teilmenge ein eindeutiges kleinstes Element besitzt.
Betrachtest du von deinen Polynomen die Grade, so ist diese Menge eine Teilmenge der natürlichen Zahlen; somit gibt es einen kleinsten Grad und damit auch ein Polynom kleinsten Grades.
Allerdings muss dieses Polynom nicht eindeutig sein, es könnte sogar unendlich viele vom kleinsten Grad geben; aber alles was du brauchst ist ja, dass es eines gibt.

lg


Bezug
                                                
Bezug
Polynom- und Hauptidealring: Mitteilung
Status: (Mitteilung) Reaktion unnötig Status 
Datum: 16:10 Mo 14.05.2012
Autor: Blackburn4717537


> Das Minimum existiert, weil die natürlichen Zahlen
> wohlgeordnet sind.
>  Das heißt (passenderweise^^), dass jede nichtleere
> Teilmenge ein eindeutiges kleinstes Element besitzt.
>  Betrachtest du von deinen Polynomen die Grade, so ist
> diese Menge eine Teilmenge der natürlichen Zahlen; somit
> gibt es einen kleinsten Grad und damit auch ein Polynom
> kleinsten Grades.
>  Allerdings muss dieses Polynom nicht eindeutig sein, es
> könnte sogar unendlich viele vom kleinsten Grad geben;
> aber alles was du brauchst ist ja, dass es eines gibt.
>  
> lg
>  

Ah ok, danke für die Erklärung. Das mit der Wohlordnung hatten wir doch schon, ganz am Anfang. Ist schon sehr sehr lange her.^^

Bezug
Ansicht: [ geschachtelt ] | ^ Forum "Lineare Algebra - Moduln und Vektorräume"  | ^^ Alle Foren  | ^ Forenbaum  | Materialien


^ Seitenanfang ^
ev.vorhilfe.de
[ Startseite | Mitglieder | Impressum ]